第II章 演習問題 [21]

各 \(\langle \alpha,n \rangle \in \kappa \times \omega\) について \(D_{\alpha,n} = \{p \in \mathbb{P} \,:\, \langle \alpha,n \rangle \in \mathrm{dom}(p)\}\) と定める. また各 \(\alpha,\beta < \kappa\), \(\alpha \neq \beta\) について \(E_{\alpha,\beta} = \{p \in \mathbb{P} \,:\, \exists n < \omega [ \langle \alpha,n \rangle \in \mathrm{dom}(p) \land \langle \beta,n \rangle \in \mathrm{dom}(p) \land p(\alpha,n) \neq p(\beta,n) ] \}\) と定める. \(|\kappa \times \omega| = |\kappa \times \kappa| = \kappa\) よりこれらは全部で \(\kappa\) 個ある. 以下, これらが条件を満たす \(\mathbb{P}\) の稠密な部分集合であることを示す.

\(\alpha < \kappa\), \(n < \omega\) のとき, \(p \in \mathbb{P} \setminus D_{\alpha,n}\) ならば \(q = p \cup \{\langle \langle \alpha,n \rangle,0 \rangle\}\) は \(q \in \mathbb{P}\) かつ \(q \leq p\) を満たす. よって \(D_{\alpha,n}\) は稠密である. また, \(\alpha,\beta < \kappa\), \(\alpha \neq \beta\) のとき, \(p \in \mathbb{P}\) とすると, \(|\mathrm{dom}(p)| < \omega\) よりある \(n < \omega\) について \(\langle \alpha,n \rangle \not\in \mathrm{dom}(p)\) かつ \(\langle \beta,n \rangle \not\in \mathrm{dom}(p)\) となる. このとき \(q = p \cup \{\langle \langle \alpha,n \rangle,0 \rangle,\langle \langle \beta,n \rangle,1 \rangle\}\) は \(q \in \mathbb{P}\) かつ \(q \leq p\) を満たす. よって \(E_{\alpha,\beta}\) も稠密である.

\(\mathbb{P}\) のフィルター \(G\) が全ての \(D_{\alpha,n}\) および \(E_{\alpha,\beta}\) と交わりを持つと仮定する. \(x \in \kappa \times \omega\), \(n,m < \omega\) かつ \(\langle x,n \rangle,\langle x,m \rangle \in \bigcup G\) のとき, \(\langle x,n \rangle \in p\) となる \(p \in G\) および \(\langle x,m \rangle \in q\) となる \(q \in G\) が存在する. このときある \(r \in G\) について \(r \leq p\) かつ \(r \leq q\) となり, \(\langle x,n \rangle \in r\) かつ \(\langle x,m \rangle \in r\), 従って ( \(r\) は関数なので) \(n = m\) となる. よって \(\bigcup G\) は (部分) 関数である. 任意の \(\alpha \in \kappa\) と \(n < \omega\) について, \(p \in G \cap D_{\alpha,n}\) を取ると \(\langle \alpha,n \rangle \in \mathrm{dom}(p) \subset \mathrm{dom}(\bigcup G)\) が成り立つ. よって \(\bigcup G\) は \(\kappa \times \omega\) から \(2\) への関数である. さらに, \(\alpha,\beta < \kappa\), \(\alpha \neq \beta\) のとき, \(p \in G \cap E_{\alpha,\beta}\) を取ると, ある \(n < \omega\) が存在して \(\langle \alpha,n \rangle,\langle \beta,n \rangle \in \mathrm{dom}(p)\) かつ \(p(\alpha,n) \neq p(\beta,n)\) が成り立つ. このとき \((\bigcup G)(\alpha,n) = p(\alpha,n) \neq p(\beta,n) = (\bigcup G)(\beta,n)\) となるので, \((\bigcup G)_\alpha \neq (\bigcup G)_\beta\) が成り立つ. 以上より主張が成り立つ.

 

解答者: 縫田光司さん (公開日: 2011年6月10日)

この解答に不具合を発見した方はぜひご指摘ください.

演習問題一覧に戻る